Đến nội dung

dinhnguyenhoangkim nội dung

Có 56 mục bởi dinhnguyenhoangkim (Tìm giới hạn từ 11-05-2020)



Sắp theo                Sắp xếp  

#545322 Giải pt $\sqrt{x+a^2}+\sqrt{x+b^2}= \...

Đã gửi bởi dinhnguyenhoangkim on 22-02-2015 - 14:56 trong Phương trình - Hệ phương trình - Bất phương trình

Cho $0< a< c< d< b$$a+b= c+d$.

Giải pt $\sqrt{x+a^2}+\sqrt{x+b^2}= \sqrt{x+c^2}+\sqrt{x+d^2}$




#545161 Chứng minh $\sum \frac{x^2+1}{y^2+1}\...

Đã gửi bởi dinhnguyenhoangkim on 21-02-2015 - 18:26 trong Bất đẳng thức - Cực trị

Cho các số thực không âm x, y, z thỏa mãn $x+y+z= 1$.

Chứng minh $\sum \frac{x^2+1}{y^2+1}\leq \frac{7}{2}$




#544819 Bộ sách về Bất đẳng thức

Đã gửi bởi dinhnguyenhoangkim on 18-02-2015 - 16:58 trong Tài nguyên Olympic toán

Em cũng không tải được




#544816 Luôn tồn tại $x,y$ thỏa mãn....

Đã gửi bởi dinhnguyenhoangkim on 18-02-2015 - 16:54 trong Số học

Tại sao các phần tử trong mỗi tập đều khác nhau theo modulo p ạ ? Anh giải thích kĩ hơn có được không ?




#544806 Tìm GTLN của $B=\frac{1}{{\sqrt{a^...

Đã gửi bởi dinhnguyenhoangkim on 18-02-2015 - 15:33 trong Bất đẳng thức và cực trị

 

Chỗ màu đỏ kia phải là 3 chứ

$6a+3b+2c=abc\Leftrightarrow \frac{2}{ab}+\frac{3}{ac}+\frac{6}{bc}=1$

Đặt $\left\{\begin{matrix} \frac{1}{a}=x & & & \\ \frac{2}{b}=y & & & \\ \frac{3}{c}=z& & & \end{matrix}\right. \Rightarrow xy+yz+xz=1$

$B=\frac{1}{\sqrt{a^2+1}}+\frac{2}{\sqrt{b^2+4}}+\frac{3}{\sqrt{c^2+9}} = \frac{1}{\sqrt{a^2+1}}+\frac{1}{\sqrt{\frac{b^2}{4}+1}}+\frac{1}{\sqrt{\frac{c^2}{9}+1}}=\frac{1}{\sqrt{\frac{1}{x^2}+1}}+\frac{1}{\sqrt{\frac{1}{y^2}+1}}+\frac{1}{\sqrt{\frac{1}{z^2}+1}}= \sum \frac{x} {\sqrt{x^2+1} }$

Lại có:$\frac{x}{\sqrt{x^2+1}}=\frac{x}{\sqrt{x^2+xy+yz+xz}}= \frac{x}{\sqrt{(x+y)(x+z)}}\leq \frac{x}{2}(\frac{1}{x+y}+\frac{1}{x+z})= \frac{1}{2}(\frac{x}{x+y}+\frac{x}{x+z})$

CMTT:$\frac{y}{\sqrt{y^2+1}}\leq \frac{1}{2}(\frac{y}{x+y}+\frac{y}{y+z})$

$\frac{z}{\sqrt{z^2+1}}\leq \frac{1}{2}(\frac{z}{x+z}+\frac{z}{y+z})$

$\Rightarrow \sum \frac{x}{\sqrt{x^2+1}}\leq \frac{1}{2}(\frac{x+y}{x+y}+\frac{y+z}{y+z}+\frac{x+z}{x+z})= \frac{3}{2}$

DBXR khi $x=y=z=\frac{1}{3}\Leftrightarrow \frac{1}{a}=\frac{2}{b}=\frac{3}{c}=\frac{1}{3}\Leftrightarrow \left\{\begin{matrix} a=3 & & & \\ b=6& & & \\ c=9& & & \end{matrix}\right.$

 

Bạn Dinh Xuan Hung nhầm chỗ dấu bằng rồi.

Dấu "=" phải là $x=y=z=\frac{1}{\sqrt3}\Leftrightarrow$$\begin{cases} & \text a=\sqrt3 \\ & \text b= 2\sqrt3\\ & \text c= 3\sqrt3 \end{cases}$




#544775 Chứng minh EF luôn qua điểm cố định.

Đã gửi bởi dinhnguyenhoangkim on 18-02-2015 - 11:04 trong Hình học

Bài này đã có rồi. Bạn vào đây xem nè. 

http://diendantoanho...nh/#entry544753




#544774 Chứng minh $(a^2+b^2, c^2+d^2)>1$

Đã gửi bởi dinhnguyenhoangkim on 18-02-2015 - 10:58 trong Số học

Hình như nhầm rồi Tùng. Phải chứng minh (c-ka) và (d-kb) đều không âm đã. Mà nếu tiếp tục thì làm sao có k=1 được. Chỗ kia là d=kb mà, mày ghi là b=kb




#544754 Chứng minh a2+b2+c2$\geq$4(a2b2+b2c2+c2a2)

Đã gửi bởi dinhnguyenhoangkim on 18-02-2015 - 08:39 trong Bất đẳng thức - Cực trị

Vậy thì sẽ là $a\to 0, b=c\to \dfrac{\sqrt{2}}{2}$ và các hoán vị.

Dấu mũi tên nghĩa là sao vậy bạn ?




#544747 Chứng minh $\sum \frac{1}{2a+1}\geq1...

Đã gửi bởi dinhnguyenhoangkim on 18-02-2015 - 01:01 trong Bất đẳng thức và cực trị

Do a, b, c > 0 và abc=1 nên tồn tại các số thực dương x, y, z sao cho  

a=$\frac{x}{y}$, b=$\frac{y}{z}$, c=$\frac{z}{x}$.

Cần chứng minh $\sum \frac{1}{\frac{2x}{y}+1}$ $\geq$ 1 .

Ta có: $\sum \frac{1}{\frac{2x}{y}+1}$=$\sum \frac{x}{2z+x}$=$\sum \frac{x^2}{2xz+x^2}$ $\geq$ $\frac{(x+y+z)^2}{x^2+y^2+z^2+2xy+2yz+2zx}$=1.




#544746 Cho tam giác ABC ... Chứng minh rằng đường thẳng EF luôn đi qua một điểm cố đ...

Đã gửi bởi dinhnguyenhoangkim on 18-02-2015 - 00:41 trong Hình học

Bạn giải cụ thể luôn được không ?




#544745 $X_aO_a;X_bO_b;X_cO_c$ đồng quy

Đã gửi bởi dinhnguyenhoangkim on 18-02-2015 - 00:38 trong Hình học

Cho $\vartriangle ABC$ có đường cao $AD,BE,CF$ đồng quy tại $H$.

Gọi $O;O_a;O_b;O_c$ thứ tự là tâm đường tròn ngoại tiếp các tam giác $ABC,AEF,DBF,DEC$.

Gọi $X,X_a,X_b,X_c$ thứ tự có cùng tọa độ tỉ cự đối với các tam giác $ABC,AEF,DBF,DEC$ sao cho $X \ne O;X_a \ne O_a; X_b \ne O_b; X_c \ne O_c$.

Chứng minh rằng: $X_aO_a;X_bO_b;X_cO_c$ đồng quy

 

Spoiler

Mình không hiều đề lắm. "tọa độ tỉ cự" là gì vậy ?




#544744 $ ACDE $ có là tg điều hoà không

Đã gửi bởi dinhnguyenhoangkim on 18-02-2015 - 00:23 trong Hình học

 

 

Thực ra bài trên là bổ đề của bài sau :

Đường tròn nội tiếp (I) của tam giác ABC tiếp xúc BC, CA, AB tại D, E, F. AD cắt (I) ở M. MB, MC cắt (I) ở P, Q. CM AD,PE,FQ đồng quy.

 

Sử dụng bổ đề rồi làm sao nữa vậy ?




#544721 Chứng minh a2+b2+c2$\geq$4(a2b2+b2c2+c2a2)

Đã gửi bởi dinhnguyenhoangkim on 17-02-2015 - 21:40 trong Bất đẳng thức - Cực trị

Sao khủng quá vậy. Con đường đi tới như thế nào vậy bạn ?




#544712 Chứng minh rằng các đường cao của tam giác $AGE, $BDK$, $...

Đã gửi bởi dinhnguyenhoangkim on 17-02-2015 - 21:12 trong Hình học

Chứng minh thế này nè em.

Gọi M, N là hình chiếu của B, J lên DK.

Khi đó, BJ vuông góc với DK$\Leftrightarrow$M$\equiv$N$\Leftrightarrow$MD2-MK2=ND2-NK2

$\Leftrightarrow$BD2-BK2=JD2-JK2.

Cái này cũng là một cách thường dùng, được sử dụng luôn đó.




#544700 $ ACDE $ có là tg điều hoà không

Đã gửi bởi dinhnguyenhoangkim on 17-02-2015 - 20:21 trong Hình học

 

Cho 6 điểm A, B, C, D, E, F trên 1 đường tròn, nằm ngược chiều kim đồng hồ theo đúng thứ tự đó. Biết rằng $ ABCD $ , $ AFED $ và $ ABDF $ là tứ giác điều hoà, hỏi $ ACDE $ có là tứ giác điều hoà không?

 

 

Áp dụng tính chất của tứ giác điều hòa và định lý Ptôlêmê vào các tứ giác nội tiếp và điều hòa nói trên,

ta được: AC.BD=2AB.CD (1)

              AF.DE=$\frac{1}{2}$AE.DF (2)

              BD.AF=AB.DF (3)

Từ (1), (2), (3) suy ra AC.DE=AE.CD $\Rightarrow$ ACDE là tứ giác điều hòa.

 




#544688 Chứng minh $\Sigma$$\sqrt{a}$$...

Đã gửi bởi dinhnguyenhoangkim on 17-02-2015 - 17:40 trong Bất đẳng thức và cực trị

Cho các số thực dương a, b, c và a+b+c$\leq$3.

Chứng minh $\Sigma$$\sqrt{a}$$\geq$$\Sigma$ab




#544677 Chứng minh rằng các đường cao của tam giác $AGE, $BDK$, $...

Đã gửi bởi dinhnguyenhoangkim on 17-02-2015 - 16:30 trong Hình học

Untitled.jpg

Gọi J là trung điểm AC.

Đặt $\widehat{ABJ}$=$\alpha$, $\widehat{CBJ}$=$\beta$.

Dễ có: AB.sin($\alpha$)=BC.sin($\beta$).

Ta sẽ chứng minh BJ vuông góc với DK.

Ta có: BJ vuông góc với DK $\Leftrightarrow$ BD2-BK2=JD2-JK2

$\Leftrightarrow$ BD2-BK2=(JB2+BD2-2JB.BD.cos($\widehat{JBD}$))-(JB2+BK2-2JB.BK.cos($\widehat{JBK}$))

$\Leftrightarrow$ BD.cos($\widehat{JBD}$)=BK.cos($\widehat{JBK}$)  $\Leftrightarrow$BD.cos($\pi$+$\alpha$)=BK.cos($\pi$+$\beta$)  $\Leftrightarrow$AB.sin($\alpha$)=BC.sin($\beta$) (đúng).

Vậy các đường cao của tam giác AGE, BDK, CIF lần lượt đi qua trung điểm của BC, CA, AB

Suy ra chúng đồng quy $\Rightarrow$ đpcm.

 




#544659 Chứng minh a2+b2+c2$\geq$4(a2b2+b2c2+c2a2)

Đã gửi bởi dinhnguyenhoangkim on 17-02-2015 - 15:17 trong Bất đẳng thức - Cực trị

Cho a, b, c > 0 và a2+b2+c2+2abc=1.

Chứng minh a2+b2+c2$\geq$4(a2b2+b2c2+c2a2)




#544641 chứng minh I,K,P,Q cùng nằm trên 1 đường tròn

Đã gửi bởi dinhnguyenhoangkim on 17-02-2015 - 12:34 trong Hình học

Untitled1.jpg

Dễ có $\widehat{PBI}$=$\widehat{PAI}$=90o nên PAIB nội tiếp. Suy ra $\widehat{APD}$=$\widehat{ABI}$

$\Rightarrow$$\widehat{APD}$=$\widehat{IBD}$.

Ta có $\widehat{APD}$=$\widehat{IBD}$, $\widehat{PDA}$=$\widehat{IDB}$

$\Rightarrow$ tam giác ADP đồng dạng với tam giác IDB

$\Rightarrow$$\frac{AD}{DP}$=$\frac{ID}{DB}$$\Rightarrow$DI.DP=DA.DB (1).

Tương tự, ta có DK.DQ=DA.DC (2).

D là trung điểm BC nên DB=DC (3).

Từ (1), (2), (3) suy ra DI.DP=DK.DQ$\Rightarrow$IKQP nội tiếp$\Rightarrow$đpcm

 

 




#544622 CM 3 đường thẳng trên đồng quy

Đã gửi bởi dinhnguyenhoangkim on 17-02-2015 - 09:50 trong Hình học

Untitled.jpg

Gọi M, P lần lượt là hinh chiếu của O lên AC, AB. N là hình ciếu của M lên AB.

Khi đó, dễ thấy d2 vuông góc với MD, d3 vuông góc với OM.

Xem BC là một trục. Giả sử D(0), A(a), E(x).

Khi đó B(-a), N($\frac{a}{2}$), P($\frac{a+x}{2}$).

Ta có:  (DO2-DD2)+(ED2-EM2)+(BM2-BO2)

          =(DO2-BO2)+(BM2-EM2)+ED2

          =(DP2-BP2)+(BN2-EN2)+ED2

         =($\frac{a+x}{2}$-0)2-($\frac{a+x}{2}$-(-a))2+($\frac{a}{2}$-(-a))2-($\frac{a}{2}$-x)2+(0-x)2=0.

Áp dụng định lý Carnot cho tam giác OMD suy ra d1, d2, d3 đồng quy

 

 

 

 




#544609 chứng minh tứ giác DEPN nội tiếp

Đã gửi bởi dinhnguyenhoangkim on 17-02-2015 - 02:03 trong Hình học phẳng

Untitled.jpg

a) Vì NE và PD là tiếp tuyến của (O) nên $\widehat{PNE}$=$\widehat{NPD}$=$\widehat{PMN}$.  

Lại có $\widehat{NPE}$=$\widehat{PND}$ $\Rightarrow$ $\widehat{DPE}$=$\widehat{END}$ 

$\Rightarrow$ PNDE nội tiếp.

b) Kẻ đường kính MH của (O), suy ra NH vuông góc với MN $\Rightarrow$ NH//PK

$\Rightarrow$ PH=NK.

Vậy MN2+NK2=MP2+PH2=MH2=4R2.




#544608 trong một mặt phẳng, có hai điểm A, B cố định, và đường thằng d song song với...

Đã gửi bởi dinhnguyenhoangkim on 17-02-2015 - 01:16 trong Hình học

Bài toán này chỉ cần A, B thuộc cùng một nửa mặt phẳng bờ d.

Gọi A' là điểm đối xứng của A qua đường thẳng d, suy ra A' cố định

Với mọi điểm M thuộc d thì MA'=MA. Ta có: MA+MB=MA'+MB$\geq$A'B không đổi.

Vậy MA+MB nhỏ nhất khi và chỉ khi M là giao điểm của A'B và đường thẳng d (với A' xác định như trên).




#544493 Topic ôn luyện cho cuộc thi toán olympic 30/4 năm 2015

Đã gửi bởi dinhnguyenhoangkim on 16-02-2015 - 16:44 trong Thi HSG cấp Tỉnh, Thành phố. Olympic 30-4. Đề thi và kiểm tra đội tuyển các cấp.

Bài 3. Cho tam giác $ABC$ nhọn khác tam giác cân nội tiếp đường tròn $\omega$. Gọi $D,E,F$ lần lượt là tâm đường tròn bàng tiếp tương ứng với đỉnh $A,B,C$ là $I$ là tâm đường tròn nội tiếp tam giác $ABC$. Đường tròn ngoại tiếp tam giác $EFI$ cắt $\omega$ tại $A_{1},A_{2}$.

$a)$ Chứng minh các đường thẳng $A_{1}A_{2}, EF, BC$ đồng quy.

$b)$ Đường tròn ngoại tiếp tam giác $DIF$ cắt $\omega$ tại $B_{1},B_{2}$; đường tròn ngoại tiếp tam giác $DIE$ cắt $\omega$ tại $C_{1},C_{2}$. Các đường thẳng $A_{1}A_{2}, B_{1}B_{2}, C_{1}C_{2}$ đôi một cắt nhau tạo thành một tam giác, chứng minh rằng diện tích tam giác nhỏ hơn $\frac{1}{4}$ diện tích tam giác $ABC$.

Untitled.jpg

a) Bằng các biến đổi góc, ta dễ có $\widehat{BFE}$+$\widehat{BCE}$=180$^{\circ}$$\Rightarrow$BCEF nội tiếp.

    Ta có BCEF nội tiếp, BA1A2C nội tiếp, FA1A2E nội tiếp $\Rightarrow$ BC là trục đẳng phương của $\omega$ và (BCE), 

    A1A2 lá trục đẳng phương của $\omega$ và (FIE), EF là trục đẳng phương của (BCE) và (FIE)

    Suy ra BC, EF, A1A2 đồng quy.

b) Dễ thấy FAIB nội tiếp.

    Ta có AB là trục đẳng phương của $\omega$ và (FAB), IF là trục đẳng phương của (FIE) và (FAB),     A     1Alà trục đẳng phương của $\omega$ và (FIE) $\Rightarrow$ AB, A  1A2, FI  đồng quy (1).

    Ta có B1B2 là trục đẳng phương của $\omega$ và (FID), IF là trục đẳng phương của (FIE) và (FID),     A     1A2  là trục đẳng phương của $\omega$ và (FIE) $\Rightarrow$ B 1B2, A1A2, IF đồng quy (2).

    Từ (1) và (2) suy ra    B1B2, A1A2, IF, AB đồng quy  $\Rightarrow$ Giao điểm của A  1A2 và B1Blà chân đường phân giác từ

    đỉnh C của $\Delta$ABC.

    Tương tự đối với A1A2 và C1C2,  C1C2 và B1B2.

    Vậy tam giác tạo bởi 3 đường thẳng   A 1A2, B1B2, C1C2 là tam giác tạo bởi chân 3 đường phân giác trong $\Delta$ABC.

    Untitled2.jpg

    Gọi các chân đường phân giác là M, N, P.

    Ta có :$\frac{SBMP}{SABC}$=$\frac{BM.BP}{BC.BA}$=$\frac{c}{b+c}$.$\frac{a}{a+b}$$\frac{ac}{(a+b)(b+c)}$

    Tương tự: $\frac{SAPN}{SABC}$=$\frac{bc}{(a+b)(c+a)}$, $\frac{SCMN}{SABC}$=$\frac{ab}{(b+c)(c+a)}$

  Vậy :$\frac{SBPM}{SABC}$+$\frac{SAPN}{SABC}$+$\frac{SCMN}{SABC}$ = $\frac{$\sum$ab(a+b)}{(a+b)(b+c)(c+a)}$=T

  Ta có: T=$\frac{(a+b)(b+c)(c+a)-2abc}{(a+b)(b+c)(c+a)}$

               $\geqslant$$\frac{(a+b)(b+c)(c+a)-(a+b)(b+c)(c+a)/4}{(a+b)(b+c)(c+a)}$=3/4

  Ta có: $\frac{SMNP}{SABC}$=1-T$\leqslant$1-$\frac{3}{4}$=$\frac{1}{4}$

   Dấu "=" không xảy ra do tam giác ABC không đều $\Rightarrow$ đpcm




#544415 Topic ôn luyện cho cuộc thi toán olympic 30/4 năm 2015

Đã gửi bởi dinhnguyenhoangkim on 16-02-2015 - 09:02 trong Thi HSG cấp Tỉnh, Thành phố. Olympic 30-4. Đề thi và kiểm tra đội tuyển các cấp.

Phép chiếu xuyên tâm bảo toàn tỉ số kép đó bạn. Cái này mình đọc trong Tài liệu chuyên toán 10 Hình.

Ở đây là phép chiếu xuyên tâm E, đi từ đường thẳng BC đến đường thẳng AG.




#544408 Topic ôn luyện cho cuộc thi toán olympic 30/4 năm 2015

Đã gửi bởi dinhnguyenhoangkim on 16-02-2015 - 08:04 trong Thi HSG cấp Tỉnh, Thành phố. Olympic 30-4. Đề thi và kiểm tra đội tuyển các cấp.

Bài 6 (Trường LHP): Cho D là điểm trên cạnh BC của tam giác ABC sao cho $\angle$CAD=$\angle$CBA. Một đường tròn tâm O đi qua B và D lần lượt cắt AB, AD tại E, F. Đường thẳng BF và DE cắt nhau tại G. M là trung điểm AG. Chứng minh CM vuông góc với AO.

 Untitled2'.jpg  

Bổ đề: Cho tứ giác ABCD nội tiếp đường tròn tâm O. AD cắt BC tại Q. AB cắt CD tại P. AC cắt BD tại I. Khi đó O là trực tâm tam giác IPQ.

Trở lại bài toán ban đầu:

Gọi H=EF$\cap$BC, K=EF$\cap$AG, J=AG$\cap$BC.

Ta có: $\angle$EFA=$\angle$ABD$\Rightarrow$$\angle$EFA=$\angle$DAC$\Rightarrow$EF//AC

Áp dụng bổ đề ta có O là trực tâm tam giác AGH $\Rightarrow$ HG vuông góc AO. Ta cần C/m CM//HG.

AJ, BF, DE đồng quy, H=EF $\cap$BC $\Rightarrow$(BDJH)=-1

$\Rightarrow$(AGJK)=-1(Phép chiếu xuyên tâm E).

Mà M là trung điểm AG nên 

$\overline{JG}$.$\overline{JA}$=$\overline{JK}$.$\overline{JM}$(Hệ thức Maclaurin)$\Rightarrow$JG.JA=JK.JM

$\Rightarrow$$\frac{JA}{JK}$=$\frac{JM}{JG}$

Lại có $\frac{JA}{JK}$=$\frac{JC}{JH}$ (do EF//HG) nên $\frac{JM}{JG}$=$\frac{JC}{JH}$$\Rightarrow$GH//MC$\Rightarrow$đpcm